Download as pdf or txt
Download as pdf or txt
You are on page 1of 79

2

Arun Singh Tomar


1. Which of the following are natural World Heritage Sites in India as mandated by
UNESCO?

1. Kaziranga National Park

2. Keoladeo National Park

3. Sundarbans National Park

4. Valley of Flowers National Park

Select the correct answer using the code given below.

(a) 1, 2, and 3 only

(b) 2 and 4 only

(c) 1, 2, 3 and 4

(d) 1, 3 and 4 only

2. The 2023 Production Gap Report titled “Phasing down or phasing up? Top fossil
fuel producers plan even more extraction despite climate promises” was recently
released. In this context, consider the following statements:

1. The report is published by the United Nations Framework Convention on Climate


Change (UNFCCC).

2. The report tracks the discrepancy between governments’ planned fossil fuel
production and global production levels consistent with limiting warming to 1.5°C or
2°C.

Which of the statements given above is/are correct?

(a) 1 only

(b) 2 only

(c) Both 1 and 2

(d) Neither 1 nor 2


3. The term 'Generative Adversarial Networks (GANs)' is often used in the context of

(a) Health care infrastructure

(b) Deepfake technology

(c) Telecommunication infrastructure

(d) Money laundering network

4. Which of the following correctly describes the theme of UN decade of 2021-30?

(a) United Nations Decade of Action on Nutrition

(b) United Nations Literacy Decade

(c) United Nations Decade for Deserts and the Fight against Desertification

(d) United Nations Decade on Ecosystem Restoration

5. Recently, India signed a MoU on “Enhancing Innovation Ecosystems through an


Innovation Handshake” with which of the following countries?

(a) Japan

(b) The United States of America

(c) The United Kingdom

(d) South Korea

6. Recently, India successfully tested fired 'Pralay' missile off Odisha coast. In this
context, consider the following statements:

1. The missile has been developed by Defence Research and Development


Organisation (DRDO).

2. It is a 350-500 km short-range, surface -to- surface missile.

3. It has a payload capacity of 500-1000 kg.

Arun Singh Tomar


How many of the above statements are correct?

(a) Only one

(b) Only two

(c) All three

(d) None

7. Fifth meeting of Conference of Parties (COP 5) to Minamata Convention held in


Geneva recently. In this context, consider the following statements:

1. The Minamata Convention on Mercury is a globally binding treaty to protect


human health and the environment from the adverse effects of mercury.

2. India has ratified the Convention.

3. In COP 5, Parties have agreed on a threshold value of 100mg/kg total


concentration of mercury in wastes contaminated with mercury.

How many of the above statements are correct?

(a) Only one

(b) Only two

(c) All three

(d) None

8. Consider the following statements with respect to the Employees' State Insurance
(ESI) Scheme:

1. The wage ceiling for coverage of employees under the ESI Act is Rs. 15,000.

2. Both employees and employers contribute equally to the contributory fund under
the ESI Scheme.

3. Employees' State Insurance Corporation (ESIC) which implements the ESI


Scheme functions under the Ministry of Finance.

Arun Singh Tomar


How many of the above statements are correct?

(a) Only one

(b) Only two

(c) All three

(d) None

9. Consider the following statements with respect to the e-way bill system under the
Goods and Services Tax (GST):

1. A consignment value of more than ₹50,000 is required to generate an e-way bill


for movement between the two States and within a State.

2. With regard to intrastate movement of goods, each state is free to choose a


threshold value for e-way bill generation.

3. No e-way bill is required for goods transported by a non-motorized conveyance.

How many of the above statements are correct?

(a) Only one

(b) Only two

(c) All three

(d) None

10. The terms 'isoniazid', 'rifampin', 'ethambutol' are often seen in the news. What
are they?

(a) synthetic chemical insecticides

(b) medications to treat tuberculosis

(c) fruit ripening hormones

(d) ozone depleting substances

Arun Singh Tomar


11. Recently, India has been removed from the Review of Significant Trade process
(RST) for Red Sanders. In this context, consider the following statements with
respect to RST and Red Sanders species:

1. The Review of Significant Trade (RST) Management System was launched by

the World Trade Organisation.

2. Red Sanders is listed in the Schedule 1 of the Wildlife Protection Act, 1972.

3. Red Sanders is endemic to South India.

How many of the above statements are correct?

(a) Only one

(b) Only two

(c) All three

(d) None

12. The 'Bletchley Declaration', sometimes seen in the news, is related to

(a) Artificial Intelligence

(b) Greenhouse gas emissions

(c) Ozone pollution

(d) Road safety

13. Recently, several state governments approached the Supreme Court over the
Governor's delay in approving the bills passed by state legislative assemblies. In this
context, consider the following statements:

1. The Constitution does not prescribe a specific timeframe for the Governor to make
a decision with respect to a bill.

Arun Singh Tomar


2. When a bill passed by the state legislature endangers the position of the state
high court, it is obligatory for the Governor to reserve the bill for the consideration of
the President.

3. If a bill is returned by the Governor and it is passed again with amendments by the
state legislature, the governor can withhold his assent to the bill.

How many statements given above are correct?

(a) Only one

(b) Only two

(c) All three

(d) None

14. Recently, Union Minister for Commerce and Industry participated in the Investor
Forum of the Partnership for Global Infrastructure and Investment (PGII). In this
context, consider the following statements:

1. It is an initiative of Group 20 (G20).

2. It is aimed to fund infrastructure projects in developing countries through public


and private investments.

Which of the above statements is/are correct?

(a) 1 only

(b) 2 only

(c) Both 1 and 2

(d) Neither 1 nor 2

15. Consider the following statements with respect to the United Nations
Commission on International Trade Law (UNCITRAL):

1. It was established in the aftermath of the dot-com bubble in the late 1990s.

Arun Singh Tomar


2. The members of the Commission are elected by the General Assembly of the UN
for a term of six years.

3. UNICTRAL is involved with state-tostate issues such as countervailing duties or


import quotas.

How many of the above statements are correct?

(a) Only one

(b) Only two

(c) All three

(d) None

16. Consider the following statements with respect to United Nations General
Assembly (UNGA):

1. In UNGA, each of the Member States of the United Nations has an equal vote.

2. The Assembly meets in regular sessions from september to december each year.

3. The UNGA elects the non-permanent members of the Security Council.

4. The Credentials Committee is appointed by the UNGA at each session.

How many of the above statements are correct?

(a) Only one

(b) Only two

(c) Onl three

(d) All four

17. Consider the following with respect to 'Gentiana kurroo', a plant mentioned in the
news recently:

1. It is endemic to Western Ghats.

Arun Singh Tomar


2. It is classified as 'critically endangered' by IUCN.

3. It is known to help treat digestive disorders and urinary infection.

How many of the above statements are correct?

(a) Only one

(b) Only two

(c) All three

(d) None

18. Consider the following countries:

1. China

2. Hong Kong

3. Russia

4. Singapore

5. United States

How many of the above-mentioned countries are members of the Asia-Pacific


Economic Cooperation (APEC)?

(a) Only two

(b) Only three

(c) Only four

(d) All five

19. Consider the following statements with respect to the Green Hydrogen Mission of
India:

1. It aims to make India a net exporter of green hydrogen.

Arun Singh Tomar


2. It targets to build capabilities to produce at least 5 million metric tonnes (MMT) of
green hydrogen per annum by 2030.

3. NITI Aayog will be responsible for the overall coordination and implementation of
the mission.

How many of the statements given above are correct?

(a) Only one

(b) Only two

(c) All three

(d) None

20. Which of the following are the components of the Global Solar Facility under
International Solar Alliance (ISA)?

1. Solar Payment Guarantee Fund

2. Solar Insurance Fund

3. Global Energy Alliance for People and Planet

4. Scaling Solar Applications for Agriculture Use

Select the correct answer using the code given below.

(a) 1 and 2 only

(b) 1, 3 and 4 only

(c) 3 and 4 only

(d) 1, 2, 3 and 4

21. Consider the following pairs:

GI Tag Product State

1. Udangudi Panangarupatti : Karnataka

Arun Singh Tomar


2. Tangsa Textile : Sikkim

3. Khamti Rice : Arunachal Pradesh

How many of the pairs given above are correctly matched?

(a) Only one

(b) Only two

(c) All three

(d) None

22. Recently, which of the following villages has been selected as Best Tourism
Village by the United Nations World Tourism Organization (UNWTO)?

(a) Dhordo

(b) Mawlynnong

(c) Malana

(d) Odanthurai

23. In the context of Nanophotonic Electron Accelerator (NEA), recently seen in the
news, consider the following statements:

1. It is the world's smallest particle accelerator which has been turned on for the first
time.

2. One of the key goals of developing NEA is to exploit the energy released by the
use of miniature particle accelerators for medicinal purposes.

3. The NEA uses light beams focused at the vacuum tube's pillars to magnify energy,
resulting in a very low energy field.

How many of the statements given above are correct?

(a) Only one

(b) Only two


Arun Singh Tomar
(c) All three

(d) None

24. With reference to 'Zero Budget Natural Farming', a term often seen in news,
which of the following statements are correct?

1. It is a method of farming done without using any credit, and without spending any
money on purchased inputs.

2. It aims to eliminate the use of chemical pesticides.

3. It is not practiced in India.

Select the correct answer using the code given below.

(a) 1 and 2 only

(b) 2 and 3 only

(c) 1 and 3 only

(d) 1, 2 and 3

25. The Worldwide Governance Indicators (WGI) is often seen in the news. It is an
initiative of

(a) Inter-Parliamentary Union

(b) Organisation for Economic Co-operation and Development

(c) World Bank

(d) United Nations

26. Consider the following statements regarding the “Loss and Damage Fund”:

1. It aims to provide financial assistance to nations most vulnerable and impacted by


the effects of climate change.

Arun Singh Tomar


2. It was opertionalised at COP-21, UN Climate Change Conference in Paris,
France.

Which of the statements given above is/are correct?

(a) 1 only

(b) 2 only

(c) Both 1 and 2

(d) Neither 1 nor 2

27. Recently, the Prime Minister launched the PM-PVTG scheme on the occasion of
Janjatiya Gaurav Divas. In this context, consider the following statements:

1. It aims to bring critical infrastructure to the 75 PVTG communities.

2. To ensure coordination it aims to appoint one nodal officer for each state.

3. The Ministry of Rural Development is the nodal ministry for overall policy planning
and coordination of programs.

How many of the statements given above are correct?

(a) Only one

(b) Only two

(c) All three

(d) None

28. The report ‘Global Landscape of Climate Finance 2023’ is released by:

(a) World Bank

(b) Intergovernmental Panel on Climate Change

(c) World Economic Forum

(d) Climate Policy Initiative

Arun Singh Tomar


29. Consider the following statements regarding Project Kusha:

1. It is aimed at developing nuclear submarines for India.

2. It is being jointly developed with the help of Israel.

Which of the statements given above is/are correct?

(a) 1 only

(b) 2 only

(c) Both 1 and 2

(d) Neither 1 nor 2

30. Recently, the World Health Organisation released the Global TB Report 2023. In
this context, consider the following statements:

1. Staphylococcus aureus is the primary causative agent of tuberculosis.

2. Tuberculosis is an infectious disease that affects only the lungs.

3. The Mantoux test is used to monitor blood sugar in Tuberculosis patients.

How many of the above statements are correct?

(a) Only one

(b) Only two

(c) All three

(d) None

31. World Heritage Outlook Report is released by:

(a) United Nations Educational, Scientific and Cultural Organization (UNESCO)

(b) Economic and Social Council (ECOSOC) of UN

(c) International Union for the Conservation of Nature (IUCN)


Arun Singh Tomar
(d) United Nations Environment Programme (UNEP)

32. Consider the following statements regarding the International Tropical Timber
Organization:

1. It is a multilateral non-governmental organization for promoting the sustainable


management and conservation of tropical forests.

2. Countries from tropical areas with timber trade are only eligible to become
members of ITTO.

3. India is one of its producing members.

How many of the above statements are correct?

(a) Only one

(b) Only two

(c) All three

(d) None

33. Consider the following statements regarding the ‘Champions of the Earth Award’:

1. It is the United Nations’ highest environmental honor.

2. It recognizes the contributions of individuals as well as organizations.

3. Recently, this award was given to laureates who are implementing solutions and
policies to eliminate plastic pollution.

How many statements given above are correct?

(a) Only one

(b) Only two

(c) All three

(d) None

Arun Singh Tomar


34. Consider the following statements regarding the Annapurna Certificate
Programme:

1. It acknowledges restaurants abroad contributing to India’s cultural cause through


cuisines.

2. It is an initiative of the Indian Council for Cultural Relations.

Which of the statements given above is/are correct?

(a) 1 only

(b) 2 only

(c) Both 1 and 2

(d) Neither 1 nor 2

35. BHARAT TAP initiative and Nirmal Jal Prayas initiative are under:

(a) Ministry of Jal Shakti.

(b) Ministry of Housing and Urban Affairs.

(c) Ministry of Environment, Forest and Climate Change.

(d) Ministry of Panchayati Raj

36. The biennial State of Worlds Forests Report is published by:

(a) World Environment Facility (WEF)

(b) United Nations Forum on Forests (UNFF)

(c) World Forest Watch (WFW)

(d) The Food and Agriculture Organization (AO)

37. Consider the following statements regarding White Phosphorous:

1. It ignites in contact with air.

Arun Singh Tomar


2. It is banned by the Convention on Certain Conventional Weapons (CCW).

Which of the statements given above is/are

correct?

(a) 1 only

(b) 2 only

(c) Both 1 and 2

(d) Neither 1 nor 2

38. The Fagradalsfjall system, recently seen in the news, is associated with?

(a) Irrigation System

(b) Volcanic system

(c) System to rehabilitate war victim

(d) System to attract honey bees

39. Consider the following statements regarding the National Organ and Tissue
Transplant Organisation (NOTTO):

1. It operates under the Ministry of Science and Technology.

2. It oversees organ procurement but distribution is left to the states.

Which of the statements given above is/are correct?

(a) 1 only

(b) 2 only

(c) Both 1 and 2

(d) Neither 1 nor 2

Arun Singh Tomar


40. Consider the following statements regarding election funding in India:

1. An individual can donate as much as he wants to a political party.

2. A company can donate up to 10% of the average of the preceding three years'
profits.

3. There is no legal expenditure limit on expenditure by political parties.

How many of the statements given above are correct?

(a) Only one

(b) Only two

(c) All three

(d) None

41. Consider the following statements regarding Just Energy Transition Partnerships
(JETPs):

1. JETPs aim to bridge the gap between developed and developing nations in
moving towards clean energy.

2. The first JETP was announced at the Sharm el-Sheikh Climate Change
Conference (COP 27).

3. South Africa, Bangladesh, and Venezuela are the first three countries to receive
funding under a JETP.

Which of the statements given above is/are correct?

(a) 1 only

(b) 1 and 2 only

(c) 2 and 3 only

(d) 3 only

42. In the context of security, 'Prachand' is a/an


Arun Singh Tomar
(a) light combat helicopter.

(b) inter-continental ballistic missile.

(c) nuclear submarine.

(d) anti-tank guided missile.

43. In the context of Indian culture, the term Vajra Mushti Kalaga was recently seen
in the news. It is a

(a) Buddhist sect

(b) form of wrestling

(c) folk dance

(d) form of puppetry

44. Recently, millions of people have been internally displaced due to ongoing
conflict with the rebel group Mouvement du 23 Mars (M23). To which country does
this M23 rebel group belong?

(a) Democratic Republic of Congo

(b) Palestine

(c) Sudan

(d) Syria

45. Consider the following statements regarding the Atmospheric Waves


Experiment:

1. It is aimed at studying the interactions between terrestrial and space weather.

2. It is an initiative of the World Meteorological Organization.

3. It will be launched and mounted on the exterior of the International Space Station
(ISS).
Arun Singh Tomar
How many of the statements given above are correct?

(a) Only one

(b) Only two

(c) All three

(d) None

46. The Palermo Convention recently seen in the news deals with

(a) transnational organized crime.

(b) sea level rise.

(c) land degradation

(d) global Financial Architecture.

47. In the context of the electoral process in India, consider the following statements
regarding the term "ENCORE":

1. It is software for complete candidate and election management for returning


officers.

2. It was developed by the National Technical Research Organisation.

Which of the statements given above is/are correct?

(a) 1 only

(b) 2 only

(c) Both 1 and 2

(d) Neither 1 nor 2

48. Consider the following statements regarding India's Carbon Credit Trading
Scheme, 2023:

Arun Singh Tomar


1. Under the scheme, MoEFCC will issue carbon credit certificates to the entities that
surpass the assigned targets.

2. The Central Electricity Regulatory Commission (CERC) will regulate the trading of
carbon credits under the scheme.

Which of the statements given above is/are correct?

(a) 1 only

(b) 2 only

(c) Both 1 and 2

(d) Neither 1 nor 2

49. Recently, India was re-elected to the Council of the International Maritime
Organization (IMO). In this context, consider the following statements:

1. International Maritime Organization is a specialized agency of the United Nations.

2. India was elected among ten states with the largest interest in providing
international shipping services.

3. India has the distinction of being a member of the IMO council continuously since
1959.

How many statements given above are

correct?

(a) Only one

(b) Only two

(c) All three

(d) None

50. Consider the following statements regarding the Global Partnership on Artificial
Intelligence (GPAI):

Arun Singh Tomar


1. It is a multistakeholder initiative to guide the responsible development and use of
artificial intelligence

2. Its secretariat is hosted by the Organisation for Economic Co-operation and


Development.

3. The first GPAI summit in 2023 was hosted by India.

How many statements given above are correct?

(a) Only one

(b) Only two

(c) All three

(d) None

51. Consider the following statements regarding the Ethics Committees of the
Parliament which was in the news recently:

1. Its origins lie in the report of the National Commission to Review the Working of
the Constitution.

2. Any person may directly complain against a member of parliament to the ethics
committee.

Which of the statements given above is/are correct?

(a) 1 only

(b) 2 only

(c) Both 1 and 2

(d) Neither 1 nor 2

52. Consider the following statements regarding alternative fuels:

1. Diesohol is an alternative fuel which is a blend between diesel and ethanol.

Arun Singh Tomar


2. A comprehensive study of ethanol blended fuel E27 was recently launched by
Hindustan Petroleum Corporation Limited (HPCL).

Which of the statements given above is/are correct?

(a) 1 only

(b) 2 only

(c) Both 1 and 2

(d) Neither 1 nor 2

53. With reference to Lithium resources, consider the following statements:

1. The lithium triangle includes Argentina, Chile, and Bolivia.

2. Recently, the Geological Survey of India discovered lithium reserves in the Reasi
district of Jammu and Kashmir.

3. ONGC Videsh, is responsible for securing lithium blocks overseas.

How many of the statements given above is/are correct?

(a) Only one

(b) Only two

(c) All three

(d) None

54. Consider the following statements with respect to the World Migration Report:

1. Since 2000, the International Organization for Migration has been publishing it
every year.

2. World Migration Report 2022 is only available online.

Which of the following statements is/are correct?

(a) 1 Only

Arun Singh Tomar


(b) 2 Only

(c) Both 1 and 2

(d) Neither 1 nor 2

55. Consider the following statements regarding Global Biofuel Alliance (GIBA):

1. It is an India-led initiative to create a favourable ecosystem for promoting


development and deployment of biofuels.

2. It was formally launched on the sidelines of the G20 Summit in New Delhi.

Which of the statements given above is/are correct?

(a) 1 only

(b) 2 only

(c) Both 1 and 2

(d) Neither 1 nor 2

56. Consider the following statements regarding Nonylphenol, sometimes seen in the
news:

1. It is a toxic bio-accumulate that is present in detergents.

2. It is an endocrine disruptor in humans and can create reproductive defects in


rodents.

3. United Nations Environment Programme has designated it as a chemical of global


concern.

4. The Bureau of Indian Standards regulates the amount of Nonyphenols in drinking


water.

How many statements given above are correct?

(a) Only one

(b) Only two


Arun Singh Tomar
(c) Only three

(d) All four

57. The term ‘JT-60SA’, which was recently seen in the news, is related to:

(a) The Japanese space mission to the sun

(b) Genetically modified millet variety developed in India

(c) Biofuel produced using jute crop

(d) Experimental nuclear fusion reactor

58. ‘Aboriginal and Torres Strait Islander Voice‘, recently seen in the news, is related
to:

(a) a movement in the Nordic countries for economic demands

(b) an organization for the protection of human rights in the states of the east coast
of the USA

(c) a referendum conducted in Australia to recognize the first people of Australia


under its constituion

(d) a movement in Caribbean islands by aboriginals for democratic rights.

59. Which of the following organizations recently launched the INFUSE mission to
study a supernova event remnant that took place 20,000 years ago?

(a) European Space Agency

(b) Japan Aerospace Exploration Agency

(c) Indian Space Research Organization

(d) National Aeronautics and Space Administration

Arun Singh Tomar


60. Consider the following statements regarding the scheme for “Development of
Solar Parks and Ultra Mega Power Projects":

1. Under the scheme, the Government of India has currently sanctioned 50 solar
parks across all the states.

2. Solar Energy Corporation of India (SECI) acts as a nodal agency for the scheme.

3. Recently, the targets for the scheme were achieved well before the expected
deadline of 2026.

How many statements given above are correct?

(a) Only one

(b) Only two

(c) All three

(d) None

61. Article 99, a rarely used provision of the United Nations Charter was invoked by
the UN Secretary General in the context of:

(a) hostilities in Gaza and Israel.

(b) Russia-Ukraine conflict.

(c) increasing hold of ISIS in employing youth in terrorism.

(d) China's growing influence in Pakistan.

62. Chief Economists Outlook report is published by

(a) World Economic Forum (WEF)

(b) The International Monetary Fund (IMF)

(c) The Organization for Economic Cooperation and Development (OECD)

(d) World Bank

Arun Singh Tomar


63. Consider the following statements with respect to the United Nations Population
Fund:

1. Its goal is to end child marriage and female genital mutilation by 2030.

2. Its funding is supported by the United Nations regular budget.

3. The UN Economic and Social Council (ECOSOC) establishes its mandate.

How many of the statements given above are

correct?

(a) Only one

(b) Only two

(c) All three

(d) None

64. India has been unanimously elected as a member representing the Asian region
in the executive committee of the Codex Alimentarius Commission (CAC), In this
context, onsider the following statements regarding the CAC:

1. It is an international food standard and code of practice contributing to safe


international food trade.

2. It was created by the Food and Agriculture Organization and the World Trade
Organization.

Which of the statements given above is/are correct?

(a) 1 only

(b) 2 only

(c) Both 1 and 2

(d) Neither 1 nor 2

Arun Singh Tomar


65. Consider the following statements with reference to 'Amaterasu particles'
recently seen in news:

1. They are high energy particles with energy exceeding 240 EeV.

2. They are produced by the Large Hadron Collider, which is the most powerful
accelerator ever built.

Which of the statements given above is/are correct?

(a) 1 only

(b) 2 only

(c) Both 1 and 2

(d) Neither 1 nor 2

66. Which of the following best describes "Operation Chakra-2" recently seen in the
news?

(a) It aimed to track down cyber criminals who indulge in financial crimes and fraud.

(b) It was launched to disrupt, degrade, and dismantle networks, with international
linkages, involved in drug trafficking.

(c) It is related to fight against the sharing and dissemination of Child Sexual Abuse
Material (CSAM) online.

(d) It aimed to reusing and recycling materials to minimize the consumption of


natural resources and prevent waste generation.

67. Which of the following statements is not correct regarding the PM-Vishwakarma
scheme?

(a) It is launched by Ministry of Micro, Small and Medium Enterprises.

(b) It recognizes artisans and craftsmen through PM Vishwakarma Certificate.

(c) Only one member per family can avail benefits of the scheme.

Arun Singh Tomar


(d) The minimum age of the beneficiary should be 14 years.

68. Recently, the State government of Gujarat banned plantation of this tree. It is a
flowering plant belonging to the family of Combretaceae. It is an invasive mangrove
species. It has been used by various public authorities in India as landscaping for
road medians, along roads, and in public gardens. This tree is famous for its dark
green leaves color throughout the year and withstands harsh environmental
conditions such as high and low temperatures.

Which of the following is best described in the passage given above?

(a) Prosopis juliflora

(b) Acacia mangium

(c) Lantana camara

(d) Conocarpus erectus

69. Mission ‘Operation Storm Makers II’ recently seen in the news is related to:

(a) Israel’s military mission in the Gaza Strip.

(b) Humanitarian assistance mission by India.

(c) INTERPOL led operation targeting human trafficking.

(d) Military exercise between India-UAE.

70. ISRO launched the ADITYA L-1 Mission to study the Sun's structure. In this
context, arrange the various layers of the Sun from inner to outer layers:

1. Convective Zone

2. Radiative Zone

3. Photosphere

4. Corona

Arun Singh Tomar


5. Chromosphere

Select the correct answer using the code given below.

(a) 3-1-5-4-2

(b) 4-5-3-1-2

(c) 2-1-3-5-4

(d) 2-3-1-5-4

71. 'Biotransformation', a new technology, recently seen in news, is related to:

(a) Production of biodegradable variety of plastic

(b) Cost-effective production of biofuels

(c) Bioremediation technolgy used in metal waste reduction

(d) Technology to use stem cells in helping disabled people

72. Consider the following statements regarding the Global Cooling Pledge:

1. It is the initiative of the United Arab Emirates as the host of CoP 28 of the
UNFCCC.

2. It commits countries to reduce coolingrelated emissions by at least 68% by 2050


compared to 2022 levels.

3. India has signed this pledge.

How many statements given above are correct?

(a) Only one

(b) Only two

(c) All three

(d) None

Arun Singh Tomar


73. The ‘State of Finance for Nature 2023’ report was released by:

(a) Global Environment Facility

(b) International Union for Conservation of Nature

(c) United Nations Environment Program

(d) Conservation International

74. Consider the following statements with reference to the Article 6A of the
Citizenship Act (1955), recently seen in news:

1. It allows people who entered India between January 1, 1966, and March 25, 1971,
and have been living in Assam to register themselves as citizens.

2. The section was inserted in the Citizenship Act in 2019.

3. Assam is the only state to have an exclusive cut-off date for citizenship against a
common one for other states.

Which of the statements given above is/are correct?

(a) 1 and 2 only

(b) 2 only

(c) 1 and 3 only

(d) 3 only

Arun Singh Tomar


Q 1.C

● Natural Heritage Sites in India, like India's Sundarbans, Western Ghats, and
Kaziranga National Park, are vital ecosystems recognised by UNESCO.
These sites conserve unique biodiversity, landscapes, and habitats. They are
a global asset, contributing to scientific research and environmental balance.
● Three types of sites: Cultural, natural and mixed.

Here's a list of the Natural Heritage sites in India, sourced from UNESCO.
● Great Himalayan National Park Conservation Area (2014)
● Kaziranga National Park (1985)
● Keoladeo National Park (1985)
● Manas Wildlife Sanctuary (1985)
● Nanda Devi and Valley of Flowers National Parks (1988, 2005)
● Sundarbans National Park (1987)
● Western Ghats (2012)
● Khangchendzonga National Park (2016) is a natural as well as cultural world
heritage site of UNESCO
● Hence option(c) is the correct answer.

Q 2.B

● The 2023 Production Gap Report: “Phasing down or phasing up? Top fossil
fuel producers plan even more extraction despite climate promises” finds that
governments plan to produce around 110% more fossil fuels in 2030 than
would be consistent with limiting warming to 1.5°C, and 69% more than would
be consistent with 2°C.
● The Production Gap Report — first launched in 2019 — tracks the
discrepancy between governments’ planned fossil fuel production and global
production levels consistent with limiting warming to 1.5°C or 2°C. Hence
statement 2 is correct.

Arun Singh Tomar


● The Report has been prepared by United Nations Environment Programme
(UNEP) and other institutes such as Stockholm Environment Institute, etc.
Hence statement 1 is not correct.
Q 3.B

● A generative adversarial network (GAN) is a class of machine learning


framework and a prominent framework for approaching generative AI. In a
GAN, two neural networks contest with each other in the form of a zero-sum
game, where one agent's gain is another agent's loss.
● Given a training set, this technique learns to generate new data with the same
statistics as the training set. For example, a GAN trained on photographs can
generate new photographs that look at least superficially authentic to human
observers, having many realistic characteristics. GANs are similar to mimicry
in evolutionary biology, with an evolutionary arms race between both
networks.
● Deepfakes are fake videos and image of people created using artificial
intelligence by swapping faces or altering voices. The deepfake technology
swaps the face of a person in a video or an image with another persons' face
through AI algorithms. The deepfake algorithm runs through a process called
Generative Adversarial Networks (GANs). Two machine learning models work
together to create forgeries and to detect them, resulting in convincing fake
videos.
● Hence option (b) is the correct answer.

Q 4.D

● The UN Decade on Ecosystem Restoration is a rallying call for the protection


and revival of ecosystems all around the world, for the benefit of people and
nature. It aims to halt the degradation of ecosystems, and restore them to
achieve global goals. Only with healthy ecosystems can we enhance people’s
livelihoods, counteract climate change, and stop the collapse of biodiversity.
● The UN Decade runs from 2021 through 2030, which is also the deadline for
the Sustainable Development Goals and the timeline scientists have identified
as the last chance to prevent catastrophic climate change.

Arun Singh Tomar


● The United Nations General Assembly has proclaimed the UN Decade
following a proposal for action by over 70 countries from all latitudes.
● Led by the United Nations Environment Programme and the Food and
Agriculture Organization of the United Nations, The UN Decade is building a
strong, broad-based global movement to ramp up restoration and put the
world on track for a sustainable future. That will include building political
momentum for restoration as well as thousands of initiatives on the ground.
● Hence, option (d) is the correct answer.

Q 5.B

● A Memorandum of Understanding (MoU) on “Enhancing Innovation


Ecosystems through an Innovation Handshake” under the framework of India
– U.S. Commercial Dialogue was signed between the two countries on the
14th of November 2023 in San Francisco. The leaders’ Joint Statement during
the historic official State Visit of Prime Minister in June 2023 announced the
establishment of the “Innovation Handshake”. Hence option (b) is the
correct answer.
● The MoU was signed at the kick-off industry roundtable titled, “Decoding the
“Innovation Handshake”: U.S. – India Entrepreneurship Partnership” in San
Francisco.
Q 6.C

● Recently, India successfully test-fired its surface-to-surface short-range


ballistic missile (SRBM) 'Pralay' from the Abdul Kalam Island off the Odisha
coast.
● The missile has been developed by the Defence Research and Development
Organisation (DRDO). Hence statement 1 is correct.
● 'Pralay' is a 350-500 km short-range, surface-to-surface missile with a
payload capacity of 500-1,000 kg. Hence statements 2 and 3 are correct.
● 'Pralay' has been developed for deployment along the Line of Actual Control
(LAC) and Line of Control (LoC). 'Pralay' missile can be compared with
China's 'Dong Feng 12' and Russia's 'Iskander' missiles.

Q 7.B

Arun Singh Tomar


• The Minamata Convention on Mercury is a global, legally binding treaty, which
was adopted in 2013 and entered into force on 16 August 2017. The core of
the Convention is protection of human health, as stated in Article 1: “to protect
human health and the environment from anthropogenic emissions and
releases of mercury and mercury compounds”. India has ratified the treaty in
2018. Hence statements 1 and 2 are correct.
• The fifth meeting of the Conference of the Parties to the Minamata
Convention (COP-5), was held in Geneva from 30 October to 3 November
2023, made significant progress by adopting 23 decisions to keep protecting
human health and the environment from the harmful effects of mercury.
Q 8.D

● The provisional payroll data of ESIC reveals that 18.88 lakh new employees
have been added in the month of September 2023. Around 22,544 new
establishments have been registered and brought under the social security
umbrella of the Employees’ State Insurance Corporation in the month of
September 2023, thus ensuring more coverage.
● ESI Scheme provides social security to the working class of the country who
come under the ambit of the ESI Scheme. The Scheme applies to factories
and establishments employing 10 or more persons in the notified areas.
Employees whose monthly wages are up to ` 21,000/- fall within the purview
of the ESI Act ( 25,000/- in the case of persons with disability). Under the ESI
Scheme, the Insured Persons are protected during contingencies such as
sickness, maternity, death or disability due to employment injury or
occupational disease. Free medical care is provided to the Insured Persons
and their family through a network of ESI Dispensaries and ESI Hospitals.
Hence statement 1 is not correct.
● The contribution payable to the ESI Corporation in respect of an employee
shall comprise of employer's contribution and the employee's contribution at a
specified rate. The rates are revised from time to time. Currently, the
employee's contribution rate is 0.75% of the wages and that of employer's is
3.25% of the wages paid/payable in respect of the employees in every wage
period. Hence statement 2 is not correct.

Arun Singh Tomar


● Employees' State Insurance Corporation (“ESIC”) is a statutory corporate
body set up under the ESI Act 1948, which is responsible for the
administration of the ESI Scheme. It functions under the Ministry of Labour
and Employment. Hence statement 3 is not correct.

Q 9.C

● E-way bill generation in October has crossed 10 crore for the first time since
its introduction. Data from the Goods and Services Tax Network (GSTN)
showed e-way bill generation touched 10.03 crore, surpassing the previous
high of 9.34 crore in August, this year.
● A waybill is a receipt or a document issued by a carrier giving details and
instructions relating to the shipment of a consignment of goods and the details
include name of the consignor, consignee, the point of origin of the
consignment, its destination, and route.
● An Electronic Way Bill (E-Way Bill) is basically a compliance mechanism
wherein by way of a digital interface the person causing the movement of
goods uploads the relevant information prior to the commencement of the
movement of goods and generates e-way bill on the GST portal.
● As per Central Goods and Services Tax (CGST) Rules, 2017, consignment
value of more than ₹50,000 is required to generate an e-way bill. This is
required for movements between the two States and within a State. However,
a State can decide the threshold for the value of goods to be applicable for
movement within its boundary. Hence statements 1 and 2 are correct.
● The validity of an e-way bill depends on the distance to be traveled by the
goods. For a distance of less than 100 Km, the e-way bill will be valid for a
day from the relevant date. For every 100 Km thereafter, the validity will be an
additional one day from the relevant date.
● No e-way bill is required to be generated in the following cases:
● Transport of goods as specified in Annexure to Rule 138 of the CGST Rules,
2017
● Goods being transported by a non-motorized conveyance. Hence statement
3 is correct.

Arun Singh Tomar


Q 10.B

● India accounts for 27 percent of the total TB cases in the world, according to
the recently released Global TB Report 2023 by the World Health
Organisation.

Key findings:
Global:
● The reported global number of people newly diagnosed with TB was 7.5
million in 2022.
● TB remained the world’s second leading cause of death in 2022 after COVID-
19.
● Net reduction of TB incidence from 2015- 2022 was 8.7% far from the WHO
End TB Strategy milestone of 50% reduction by 2025.
● India’s findings
● India, Indonesia, and the Philippines collectively account for nearly 60% of the
reduction in number of people newly diagnosed with TB in 2020- 2021.
● India has 27% of the world’s TB cases.
● TB is caused by bacillus Mycobacterium tuberculosis which most often affects
the lungs (pulmonary TB). The most common medications to treat TB include
isoniazid, rifampin, ethambutol, pyrazinamide, etc. Currently, Bacillle
Calmette-Guerin is the only licensed vaccine available for TB prevention. It
spreads from person to person through air. Hence option (b) is the correct
answer.
● TB Risk factors: Diabetes, HIV infection, Undernutrition, tobacco use.

Q 11.A

● Based on our compliance and reporting, India has been removed from the
Review of Significant Trade for Red Sanders. India was under Review of
Significant Trade (RST) process for Red Sanders since 2004.
● At CoP19, the CITES Secretariat launched the Review of Significant Trade
(RST) Management System, developed by the United Nations International

Arun Singh Tomar


Computing Centre and funded by the European Union. Hence statement 1 is
not correct.
● RST is a process through which the CITES Standing Committee places
increased scrutiny on the exports of a species from a country to determine if
the Convention is being properly implemented. In the past, it has even led to a
recommendation to suspend trade with India. Removing from the RST list is a
major boost for the farmers who grow Red Sanders.
● Red sanders (Pterocarpus santalinus) is a high market value tree, endemic to
few districts in Andhra Pradesh in southern India. The species has been listed
as Appendix II under CITES since 1994. It is listed under Schedule IV of the
Wildlife Protection Act, 1972. Hence statement 2 is not correct and
statement 3 is correct.

Q 12.A

● Calling for international cooperation on ensuring safe use of Artificial


Intelligence (AI), the Britain government published the ‘Bletchley Declaration
agreement with countries like India, Australia, China, and the US among
others, on November 1, the first day of the UK AI summit. Hence option (a) is
the correct answer.
Q 13.B

● Tamil Nadu Governor has withheld his assent to a batch of 10 Bills, even as
at least four opposition-ruled states are before the SC seeking relief from the
increasingly common practice of Raj Bhavans halting the elected
government’s lawmaking process.
● While Article 163 of the Constitution deals with the powers of the Governor
generally, Article 200 specifically deals with the issue of granting assent to
Bills. Both the provisions are read together to determine the contours of the
power the Governor holds on this issue.
● When a bill is sent to the governor after it is passed by state legislature, he
can:
● Give his assent to the bill, or
● Withhold his assent to the bill, or

Arun Singh Tomar


● Return the bill (if it is not a money bill) for reconsideration of the state
legislature. However, if the bill is passed again by the state legislature with or
without amendments, the governor has to give his assent to the bill. Hence
statement 3 is not correct.
● Reserve the bill for the consideration of the president. In one case such
reservation is obligatory, that is, where the bill passed by the state legislature
endangers the position of the state high court. Hence statement 2 is correct.
● In addition, the governor can also reserve the bill if it is of the following nature:
● Ultra-vires, that is, against the provisions of the Constitution.
● Opposed to the Directive Principles of State Policy.
● Against the larger interest of the country.
● Of grave national importance.
● Dealing with compulsory acquisition of property under Article the Constitution.
● However, the Constitution does not prescribe a time limit for the Governor to
assent, withhold assent or reserve the Bill for the consideration of the
President. Hence statement 1 is correct.

Q 14.B

● Union Minister for Commerce and Industry, Consumer Affairs, Food and
Public Distribution and Textiles, Shri Piyush Goyal, participated in the Investor
Forum of the Partnership for Global Infrastructure and Investment (PGII) &
Indo-Pacific Economic Framework for Prosperity (IPEF).
● The Partnership for Global Infrastructure and Investment (PGII) is a
collaborative effort by Group of Seven(G7) to fund infrastructure projects in
developing nations based on the trust principles of the Blue Dot Network.
Hence statement 1 is not the correct answer.
● It is considered to be the bloc's counter to China's Belt and Road Initiative and
a key component of the "Biden Doctrine". The partnership plan was
announced for the first time in June 2022 during the 48th G7 summit in
Germany.
● The Partnership for Global Infrastructure and Investment (PGII) is a shared
G7 commitment to advance public and private investments in sustainable,
inclusive, resilient and quality infrastructure. Through this partnership, the G7
aims to mobilize up to USD 600 billion by 2027 in order to narrow the
Arun Singh Tomar
infrastructure investment gap in partner countries. Hence statement 2 is
correct.
Q 15.A

● The district court of The Hague, in the Netherlands, has rejected a plea by the
Indian government to set aside a compensation award of $111 million made
by a tribunal in favour of foreign investors in the Bengaluru satellite services
start-up firm Devas Multimedia over the cancellation of a 2005 satellite deal
with ISRO’s Antrix Corporation in 2011.
● Three Mauritius-based investors in Devas Multimedia were awarded $111
million in compensation by the United Nations Commission on International
Trade Law (UNCITRAL) tribunal in The Hague on October 13, 2020, over the
failed Devas-Antrix satellite deal.
● The United Nations Commission on International Trade Law is the core legal
body of the United Nations system in the field of international trade law. The
United Nations Commission on International Trade Law (UNCITRAL) was
established by the General Assembly in 1966. Hence statement 1 is not
correct.
● The Commission carries out its work at annual sessions, which are held in
alternate years at United Nations Headquarters in New York and at the
Vienna International Centre at Vienna. Each working group of the
Commission typically holds one or two sessions a year, depending on the
subject matter to be covered; these sessions also alternate between New
York and Vienna.
● It has 70 members. The membership is representative of the various
geographic regions and the principal economic and legal systems of the
world. Members of the Commission are elected by the General Assembly for
terms of six years, the terms of half the members expiring every three years.
Hence statement 2 is correct.
● The issues dealt with by the WTO and UNCITRAL are different. The WTO
deals with trade policy issues, such as trade liberalization, abolition of trade
barriers, unfair trade practices or other similar issues usually related to public
law, whereas UNCITRAL deals with the laws applicable to private parties in
international transactions. As a consequence, UNCITRAL is not involved with

Arun Singh Tomar


"state-to-state" issues such as anti-dumping, countervailing duties, or import
quotas. Hence statement 3 is not correct.

Q 16.D

● Recently, the UN General Assembly voted by a large margin against the US'
economic and trade embargo against Cuba. A total of 187 states voted for the
resolution put forward against the embargo with only the US and Israel voting
against and Ukraine abstaining.
● Established in 1945 under the Charter of the United Nations, the General
Assembly occupies a central position as the chief deliberative, policymaking
and representative organ of the United Nations. Comprised of all 193
Members of the United Nations, it provides a unique forum for multilateral
discussion of the full spectrum of international issues covered by the Charter.
Each of the 193 Member States of the United Nations has an equal vote.
Hence statement 1 is correct.
● The UNGA also makes key decisions for the UN, including:
● Appointing the Secretary-General on the recommendation of the Security
Council o electing the non-permanent members of the Security Council
● Approving the UN budget. Hence statement 3 is correct.
● The Assembly meets in regular sessions from September to December each
year, and thereafter as required. It discusses specific issues through
dedicated agenda items or sub-items, which lead to the adoption of
resolutions. Hence statement 2 is correct.
● A Credentials Committee is appointed at the beginning of each regular
session of the General Assembly. It consists of nine members, who are
appointed by the General Assembly on the proposal of the President. The
Committee reports to the Assembly on the credentials of representatives.
Hence statement 4 is correct.

Q 17.B

● Recently the plant 'Gentiana Kurroo' was in news as the efforts of experts in
the research wing of the Uttarakhand Forest Department have brought back
the 'critically endangered' plant from the brink of extinction. Endemic to the
Western Himalayas, Gentiana Kurroo, commonly known as Himalayan

Arun Singh Tomar


Gentian or Trayman, is a unique and revered medicinal herb. Hence
statement 1 is not correct.
● The highly valued plant holds a rich history in traditional medicine and has
promising prospects in modern healthcare - it is known to help treat liver
ailments digestive disorders, diabetes, bronchial asthma, and urinary
infection. The remarkable feature of this plant is its distinctive vibrant, trumpet
shaped blue flowers (the presence of blue-colored flowers in angiosperms is
relatively uncommon). Hence statement 3 is correct.
● These flowers typically bloom from mid-September to October and feature a
characteristic white or yellow spot at their base. The root of the Himalayan
Gentian is known for its therapeutic properties - particularly in treating liver
ailments. As a result, it has always been subjected to overexploitation, taking
it to the verge of extinction.
● Classified as 'critically endangered' by the International Union for
Conservation of Nature (IUCN), the Himalayan Gentian is one of the 16 plant
species in the Uttarakhand Biodiversity Boardlist of 'threatened species' in
the state. Hence statement 2 is correct.

Q 18.D

● The latest edition (30th) of the Asia-Pacific Economic Cooperation (APEC)


Economic Leaders' Meeting was concluded in San Francisco with the Golden
Gate Declaration, in which the leaders underscored their commitment to
"Creating a Resilient and Sustainable Future for All."
● The Asia-Pacific Economic Cooperation (APEC) is a regional economic forum
established in 1989 to leverage the growing interdependence of the Asia-
Pacific. APEC's 21 members aim to create greater prosperity for the people of
the region by promoting balanced, inclusive, sustainable, innovative and
secure growth and by accelerating regional economic integration.
● The 21 members of the APEC are Australia, Brunei, Canada, Chile, China,
Hong Kong, Indonesia, Japan, South Korea, Malaysia, Mexico, New Zealand,
Papua New Guinea, Peru, Philippines, Russia, Singapore, Taiwan, Thailand,
United States, Vietnam. Hence option (d) is the correct answer.
Q 19.B

Arun Singh Tomar


● Union Government of India approved the National Green Hydrogen Mission
with an outlay of ₹ 19,744 crore from FY 2023-24 to FY 2029-30. The
overarching objective of the Mission is to incentivize the commercial
production of green hydrogen and make India a net exporter of the fuel. The
Mission will facilitate demand creation, production, utilization and export of
Green Hydrogen. Hence, statement 1 is correct.
● The Mission will have wide-ranging benefits- the creation of export
opportunities for Green Hydrogen and its derivatives; Decarbonisation of
industrial, mobility and energy sectors; reduction in dependence on imported
fossil fuels and feedstock; development of indigenous manufacturing
capabilities; creation of employment opportunities; and development of
cutting-edge technologies. It aims to achieve the above objectives, the
Mission will build capabilities to produce at least 5 Million Metric tonnes
(MMT) of Green Hydrogen per annum by 2030, with the potential to reach 10
MMT per annum with the growth of export markets. The Mission will support
the replacement of fossil fuels and fossil fuel-based feedstocks with
renewable fuels and feedstocks based on Green Hydrogen. Hence,
statement 2 is correct.
● Achievement of Mission targets is expected to reduce a cumulative ₹ 1 lakh
crore worth of fossil fuel imports by 2030.
● The Ministry of New and Renewable Energy (MNRE) will be responsible for
the overall coordination and implementation of the Mission. The Mission
Secretariat, headquartered in MNRE, will formulate schemes and programs
for financial incentives to support the production, utilization and export of
Green Hydrogen and its derivatives. Hence, statement 3 is not correct.

Q 20.A

● The International Solar Alliance (ISA) announced that the Global Solar Facility
(GSF), formed by it to stimulate investments into solar power projects, is set
to receive a capital contribution of $35 million. The Indian government is
considering a $25 million investment as capital contribution in the GSF in
addition to $10 million coming from the ISA.
● It is a payment guarantee mechanism expected to stimulate investments into
solar projects, with two financial components:-
Arun Singh Tomar
● Solar Payment Guarantee Fund to provide a partial guarantee and enable
investments in geographies that do not receive investments.
● Solar Insurance Fund to reduce the burden of insurance premium for solar
developers in prerevenue phase of project.
● Initiatives taken by ISA but not under Solar Facility:-
● Green Grids Initiative - One Sun, One World, One Grid (OSOWOG).
● Global Energy Alliance for People and Planet (GEAPP) launched at COP26
with USD10 billions.
● ISA’s Programme on Scaling Solar Applications for Agriculture Use (SSAAU).
● Hence option (a) is the correct answer.

Q 21.A

● Udangudi ‘Panangkarupatti’ (palm jaggery/ gur) from Tamil Nadu has been
given a Geographical Indication (GI) tag.
● The palm jaggery preparation procedure in this area is traditional to date
without the inclusion of any additional modern strategies.
● Triple superphosphate and phosphoric acid are used in many other areas, but
no such chemical additives are used in Udangudi Panangkarupatti. Hence
pair 1 is not correctly matched.
● The karupatti prepared from the palm sap from the region around Udangudi in
Tiruchendur taluk in Thoothukudi district has some uniqueness. This is due to
the presence of red sand dune soil found in the region.
● This soil holds less groundwater. The moisture content in the atmosphere is
less because of the dry climatic conditions, which leads to high sucrose
content, in turn adding taste.
● Khamti rice is a variety of sticky rice produced in the Namsai district of
Arunachal Pradesh and is known for its taste. It is cultivated by traditional
Khampti tribal farmers. Hence pair 3 is correctly matched.
● Tangsa textile is made by the Tangsa tribe of Changlang district in Arunachal
Pradesh and is famous for its exotic designs and colors. Hence pair 2 is not
correctly matched.
● The first ever yak milk product, Arunachal Pradesh Yak Churpi also received
a GI tag recently.

Arun Singh Tomar


Q 22.A

● Recently, in 2023 the Dhordo village in the Kutch district of Gujarat was
awarded as the Best Tourism Village by the United Nations World Tourism
Organization (UNWTO).
● From Salty marshland to an Iconic destination of the Western Indian
Subcontinent – Dhordo, in the Rann of Kutch, has become the face of
Gujarat’s development.
● Tourism was launched through Rann Utsav, a four-month festival whereby a
special tent city was created in the desertic land with all the necessary
amenities made available to tourists.
● It is during these four months that the village comes to life displaying all the
natural and cultural resources/products to its consumers.
● Mawlynnong is a village in the East Khasi Hills district of the Meghalaya state
in North East India. It is notable for its cleanliness and also was chosen by
Discover India magazine as Asia's cleanest village.
● The remote village of Malana is nestled in the pristine Parvati Valley of
Himachal Pradesh, and stands as a mysterious and enchanting destination.
Known for its unique customs, centuries-old traditions, and breathtaking
natural beauty, Malana is a place that has captured the imaginations of
travelers for years.
● Odanthurai of Tamil Nadu has taken energy generation to another level. The
village not only produces its own electricity but also sells it to the state’s
government. The village has come up with self-help ventures Wind-power
generation.
● They have replaced the grid electricity with a 9kW biomass gasifier power
generation system for the means of pumping water to the houses.
● Hence option (a) is the correct answer.

Q 23.C

● Recently, Scientists have made history by successfully turning on the world's


smallest particle accelerator for the first time. With a size similar to a small
coin, this astonishing discovery holds enormous potential for a variety of

Arun Singh Tomar


applications, including the use of miniature particle accelerators for medicinal
purposes. Hence statements 1 and 2 are correct.
● The NEA is a cutting-edge technology made up of a microscopic
microprocessor and an even smaller vacuum tube made up of thousands of
individual pillars.
● By directing microscopic laser beams onto these tiny pillars, researchers have
developed a way for accelerating electrons. Surprisingly, the main
acceleration tube within the NEA is only 0.02 inches (0.5 millimeters) long,
which is 54 million times shorter than the massive 16.8-mile-long (27-
kilometer) ring housing CERN's Large Hadron Collider (LHC) in Switzerland,
the world's largest and most powerful particle accelerator.
● Nano-Scale Precision: The interior of the NEA's tiny tunnel is around 225
nanometers broad. According to the National Nanotechnology Institute, the
thickness of a human hair ranges between 80,000 and 100,000 nanometers.
● Acceleration of Electrons: Researchers from the Friedrich-Alexander
University of Erlangen-Nuremberg (FAU) in Germany used the NEA to
accelerate electrons in a recent study published in the journal Nature. They
effectively increased electron energy from 28.4 kiloelectron volts (keV) to 40.7
keV, a 43% increase. This is the first successful activation of a nanophotonic
electron accelerator, which was first proposed in 2015.
● It is a Particle Acceleration on a Microchip.
● In contrast to Large Particle Accelerators: The LHC uses almost 9,000
magnets to create a magnetic field for particle acceleration, whereas the NEA
uses light beams focused at the vacuum tube's pillars to magnify energy,
resulting in a far lower energy field. Electrons accelerated by the NEA have
substantially less energy than electrons accelerated by major colliders such
as the LHC. However, experts believe that by experimenting with different
materials or stacking many tubes together, they can improve particle
acceleration even further. Despite this, the NEA will not achieve energy levels
equivalent to those of massive particle colliders. Hence statement 3 is
correct.
Q 24.A

Arun Singh Tomar


● Statement 1 is correct. ‘Zero budget’ farming promises to end a reliance on
loans and drastically cut production costs, ending the debt cycle for desperate
farmers. The word ‘budget’ refers to credit and expenses, thus the phrase
'Zero Budget' means without using any credit, and without spending any
money on purchased inputs.
● Statement 2 is correct. The main aim of ZBNF is eliminate use of chemical
pesticides and uses biological pesticides and promote of good agronomic
practices. Farmers use earthworms, cow dung, urine, plants, human excreta
and such biological fertilizers for crop protection.
● Statement 3 is not correct. It has attained wide success in southern India,
especially the southern Indian state of Karnataka where it first evolved. The
movement in Karnataka state was born out of collaboration between Mr
Subhash Palekar, who put together the ZBNF practices, and the state farmers
association Karnataka Rajya Raitha Sangha (KRRS).

Q 25.C

● Recent context : Raising concerns over the use of World Bank’s Worldwide
Governance Indicators in ratings assessment by credit ratings agencies,
especially for emerging economies, Chief Economic Adviser V Anantha
Nageswaran said there is a need for the World Governance Index to be more
transparent and less subjective.
● The World Bank’s Worldwide Governance Indicators provide a ranking of 215
countries territories based on six dimensions of governance: ‘Voice and
Accountability’; ‘Political Stability and Absence of Violence’; ‘Government
Effectiveness’; ‘Regulatory Quality’; ‘Rule of Law’ and ‘Control of Corruption.
● The WGI were developed in 1999 by two World Bank researchers, Daniel
Kaufmann and Aart Kraay. The data are updated annually each September.
● The WGI aggregate data from more than 30 think tanks, international
organizations, nongovernmental organizations, and private firms across the
world selected on the basis of three key criteria:

1) they are produced by credible organizations;

2) they provide comparable cross-country data;

3) they are regularly updated.


Arun Singh Tomar
● Hence option (c) is the correct answer.

Q 26.A

● Recently, at COP28, 196 parties adopted the decision to operationalise the


Loss and Damage fund. Hence statement 2 is not correct.
● Loss and damage refers to the negative consequences that arise from the
unavoidable risks of climate change, like rising sea levels, prolonged
heatwaves, desertification, the acidification of the sea and extreme events,
such as bushfires, species extinction and crop failures.
● The fund was set up during COP27 at Sharm el-Shaikh in Egypt, 2022. The
establishment of a Loss and Damage Fund was, for many, the highlight of the
United Nations Climate Conference (COP 27) and the culmination of decades
of pressure from climate-vulnerable developing countries.
● The fund aims to provide financial assistance to nations most vulnerable and
impacted by the effects of climate change. Hence statement 1 is correct.

Q 27. A

● Recently, the Prime Minister launched the ₹24,000-crore Pradhan Mantri


Particularly Vulnerable Tribal Groups (PM PVTG) Mission aimed at the holistic
development of around 28 lakh primitive tribals spread across 220 districts
across the country, on the occasion of tribal icon Birsa Munda’s birth
anniversary and the third “Janjatiya Gaurav Diwas.
● The mission is aimed to improve the socio-economic conditions of the 75
particularly vulnerable tribal groups (PVTGs) by saturating PVTG families and
habitations with basic facilities such as safe housing, clean drinking water,
and sanitation, improved access to education, health and nutrition, road and
telecom connectivity, and sustainable livelihood opportunities. Hence
statement 1 is correct.
● PVTGs are characterized by a “pre-agriculture level of technology, stagnant or
declining population, extremely low literacy, and subsistence level of the
economy. PVTGs were recognized as a separate category based on the
findings of the 1961 Dhebar Commission.
● To ensure coordination and effective implementation, The ministry has
appointed 1 nodal officer for each PVTG community and they are visiting their

Arun Singh Tomar


habitations to understand their requirements. Hence statement 2 is not
correct.
● The Ministry of Tribal Affairs is the Nodal Ministry for overall policy planning
and coordination of programs for the development of STs. Hence statement
3 is not correct.
● Hence option (a) is the correct answer.

Q 28.D

● Climate finance is one of the key agenda items up for discussion at the much-
awaited 28th Conference of Parties (COP28) to the United Nations
Framework Convention on Climate Change to be held in Dubai later this
month.
● Climate Policy Initiative’s (CPI) new report Global Landscape of Climate
Finance 2023 brings together the latest data and analysis in this regard.
● The report breaks down the flow of climate finance by its application,
geographical distribution and sources, using data from 2021 and 2022. The
flows tracked in the report “represent targeted climate mitigation and
adaptation-specific project-level allocation of capital”
● The Climate Policy Initiative: It is an independent non-profit research group
and international climate policy organization based in San Francisco,
California with other offices worldwide. CPI is supported primarily by
philanthropic organizations and government development finance.

Q 29.B

● The Government of India has given the approval for "Acceptance of


Necessity" for Project Kusha recently.
● Project Kusha focuses on the development of Long-Range Surface-to-Air
Missiles (LR-SAM) and ultimately an air defense system for India. Hence
statement 1 is not correct.
● It is being jointly developed with Israel Aerospace Industries, Israel’s major
aerospace and aviation manufacturers. Hence statement 2 is correct.

Arun Singh Tomar


● The mobile LR-SAM, with its long-range surveillance and fire control radars,
would have different types of interceptor missiles designed to hit hostile
targets at 150 km, 250 km, and 350 km ranges.
● LR-SAM isn’t the only project India is developing with Israel. India’s Kalyani
Strategic Systems has already entered a joint venture with Israel’s Rafael
Advanced Systems Ltd for indigenous production of Medium-Range Surface
to Air Missile (MRSAM).

Q 30.D

● Tuberculosis, often called TB, is a bacterial infection that primarily affects the
lungs. It's caused by Mycobacterium tuberculosis and usually spreads through
the air when an infected person coughs or sneezes. Symptoms include
coughing, chest pain, fatigue, weight loss, and fever.
● Mycobacterium tuberculosis is the primary agent responsible for TB and not
Staphylococcus aureus. Hence statement 1 is not correct.
● The bacteria usually attack the lungs, but TB bacteria can attack any part of
the body such as the kidney, spine, and brain. Not everyone infected with TB
bacteria becomes sick. As a result, two TBrelated conditions exist: latent TB
infection (LTBI) and TB disease. Hence statement 2 is not correct.
● The Mantoux tuberculin skin test (TST) is one method of determining whether
a person is infected with Mycobacterium tuberculosis, and not to monitor
sugar level. Hence statement 3 is not correct.
Q 31.C

● The World Heritage Outlook Report is the flagship assessment of the status of
natural World Heritage sites by the International Union for Conservation of
Nature (IUCN). The report is produced every 3 years and provides an
overview of the conservation status of all natural World Heritage sites, as well
as trends and challenges. Hence option (c) is the correct answer.
Q 32.A

● The International Tropical Timber Organization (ITTO) is an intergovernmental


organization that promotes the sustainable management and conservation of
tropical forests and the expansion and diversification of international trade in
Arun Singh Tomar
tropical timber from sustainably managed and legally harvested forests.
Hence statement 1 is not correct.
● It was established in 1986 under the International Tropical Timber Agreement
(ITTA), which was sponsored by the United Nations Conference on Trade and
Development (UNCTAD) and ratified in 1985.

The main objectives of the ITTO are to:


● Develop internationally agreed policy guidelines and norms to encourage
sustainable forest management (SFM) and sustainable tropical timber
industries and trade.
● Assist tropical member countries in adapting such guidelines and norms to
local circumstances and to implement them in the field through projects and
other activities.
● Collect, analyze, and disseminate data on the production and trade of tropical
timber.
● Members:
● ITTO divides its members into two categories - Producer Countries and
Consumer Countries.
● Producer Countries:
● Producer countries are countries that are located between the Tropic of
Cancer and the Tropic of Capricorn, have tropical forest resources, and/or are
net exporters of tropical timber in volume terms.
● These countries are typically developing countries that rely on the export of
tropical timber products for a significant portion of their income.
● There are 37 Producer Countries which include Bolivia, Nigeria, India, etc.
Hence statement 3 is correct.

Consumer Countries:
● Consumer countries are countries that are located outside of the tropics and
generally import more primary tropical wood products than they export. These
countries are typically developed countries that have a high demand for
tropical timber products.
● There are 38 Consumer countries including Albania, Australia, China, etc.

Arun Singh Tomar


● The ITTO is open to all countries, regardless of whether they have tropical
forests or engage in the timber trade. As of today, the ITTO has 156 member
countries, including developed and developing countries with and without
tropical forests. Hence statement 2 is not correct.

Q 33.C

● Champions of the Earth award honour individuals, groups, and organizations


whose actions have transformative impact on environment. Hence statement
2 is correct.
● Awarded annually since its inception in 2005, it is UN’s highest environmental
honour which recognizes outstanding leaders from government, civil society
and private sector. Hence statement 1 is correct.
● To eliminate plastic pollution, 2023 award recognized in four categories.
Hence statement 3 is correct.
Q 34.C

● Six Indian restaurants from around the world serving desi cuisine have been
chosen for this year’s Annapurna Certificate Programme.
● It is an initiative by the Indian Council for Cultural Relations (ICCR) to
recognize Indian restaurants abroad that are contributing to the country’s
cultural cause in the culinary way. Hence both statements 1 and 2 are
correct.
Q 35.B

● BHARAT TAP initiative and Nirmal Jal Prayas initiative are under the Ministry
of Housing and Urban Affairs.
● Focus: Promotes low-flow fixtures and sanitaryware to reduce water usage.
● Goal: Aim to achieve a minimum 40% reduction in water consumption through
taps, showers, and other fittings.
● Implementation: Encourages the use of Bureau of Indian Standards (BIS)
certified water-efficient plumbing fixtures.

Arun Singh Tomar


● Benefits: Reduced water usage, lower water bills, lessened strain on water
resources, and potentially decreased energy consumption for water treatment
and pumping.
● Launched by: Ministry of Housing and Urban Affairs (MoHUA) in collaboration
with the Indian Plumbing Association (IPA).

Nirmal Jal Prayas:


● Focus: Tackles wastewater management and promotes sustainable water
reuse.
● Goal: Aim to save 500 crore litres of water per year through wastewater
treatment and reuse.
● Implementation: Supports development and adoption of wastewater treatment
technologies, encourages rainwater harvesting, and promotes public
awareness campaigns.
● Benefits: Reduced reliance on freshwater sources, improved sanitation and
hygiene, potential for agricultural irrigation or industrial use of treated
wastewater.
● Launched by: National Real Estate Development Council (NAREDCO)
through its MAHI (Municipal Authorities Helping Infrastructure) initiative.
● The National Real Estate Development Council(NAREDCO) was established
as an autonomous self regulatory ody in 1998 under the aegis of the Ministry
of Housing and Urban Affairs.
● Hence, option (b) is the correct answer.

Q 36.D

● The State of the World's Forests Report: The State of the World's Forests
Report, is a flagship publication of the Food and Agriculture Organization of
the United Nations (FAO). It is published every two years and provides a
comprehensive overview of the state of the world's forests. The recent
publication is released in 2022 which focuses on the potential of forests to
contribute to green recovery and a transition to sustainable economies.
Hence option (d) is the correct answer.
Q 37.A

Arun Singh Tomar


● Lebanon accuses Israel of white phosphorus attacks.
● White phosphorus is a chemical waxy solid substance typically appearing
yellowish or colorless, and some have described its odor as resembling garlic.
● It ignites instantly upon contact with air. Hence statement 1 is correct.
● It is often used by militaries to illuminate battlefields, generate a smokescreen,
and as an incendiary.
● Once ignited, white phosphorus is very difficult to extinguish. It sticks to
surfaces like skin and clothing.
● White phosphorus is harmful by all routes of exposure.
● White phosphorus can cause deep and severe burns, penetrating even
through bone, and has been known to reignite after initial treatment. After
exposure, the priority is to stop the burning process.
● Convention on Certain Conventional Weapons (CCW) imposes restrictions on
the use of white phosphorus, to safeguard civilians. It is not banned. Hence
statement 2 is not correct.
Q 38.B

● The Fagradalsfjall system is related to the volcanic system. It is situated on


the Reykjanes Peninsula in Iceland, a region known for its remarkable
geological and geothermal features. It is the “world’s newest baby volcano.” It
had been dormant for eight centuries before erupting in 2021, 2022 and 2023.
● The Reykjanes Peninsula is located in the southwestern part of Iceland and is
known for its proximity to the Mid-Atlantic Ridge, a tectonic boundary where
the North American and Eurasian tectonic plates meet. This geological setting
makes the Reykjanes Peninsula a hub of dynamic volcanic and geothermal
activity. The eruption marked the first time in over 800 years that a volcanic
event had occurred in this particular area. The eruption is characterized by its
effusive style, where molten lava flows out relatively calmly compared to more
explosive volcanic eruptions.
● Deep under the Earth’s surface, intense heat melts rocks to form magma, a
thick flowing substance lighter than solid rock. This drives it upwards and
most of it gets trapped in magma chambers deep underground. Over time,
this viscous liquid cools and solidifies once again. However, a tiny fraction
erupts through vents and fissures on the surface, causing volcanic eruptions.

Arun Singh Tomar


The movement of magma close to the Earth’s surface exerts a force on the
surrounding rock, which often causes earthquake swarms.
● Hence option (b) is the correct answer

Q 39.D

● National Organ and Tissue Transplant Organisation data shows four out of
five organ recipients in the country are men highlighting gender disparity.
● National Organ and Tissue Transplant Organization (NOTTO) is a National
level organization set up under the Directorate General of Health Services,
Ministry of Health and Family Welfare. It has the following two divisions :
● National Human Organ and Tissue Removal and Storage Network
● National Biomaterial Centre
● Hence statement 1 is not correct.
● National Organ and Tissue Transplant Organization functions as an apex
center for All India activities of coordination and networking not just for
procurement but the distribution of Organs and Tissues and registry of Organs
and Tissues Donation and Transplantation in the country. Hence statement 2
is not correct.

Q 40.B

● Recently a 5 judge bench of the Supreme Court has started hearing the
petitioner's case challenging the constitutional validity of electoral bonds.
● Electoral bonds are interest-free bearer bonds or money instruments that can
be purchased by companies and individuals in India from authorized branches
of the State Bank of India (SBI).
● These bonds are sold in multiples of Rs 1,000, Rs 10,000, Rs 1 lakh, Rs 10
lakh, and Rs 1 crore. They can be purchased through a KYC-compliant
account to make donations to a political party. The political parties have to
encash them within a stipulated time.
● The name and other information of the donor are not entered on the
instrument and thus electoral bonds are said to be anonymous. There is no
cap on the number of electoral bonds that a person or company can
purchase.
Arun Singh Tomar
● Political parties that secured at least 1% of the votes polled in the recent Lok
Sabha or State Assembly elections and are registered under the RPA can get
a verified account from the Election Commission of India (ECI). The bond
amounts are deposited in this account within 15 days of their purchase.
● The political party has to encash the amount within those 15 days, and the
amount received as a donation gets deposited into the Prime Minister's Relief
Fund. These bonds, however, are not available for purchase all the time.
● In India, there are no donation limits on individuals. Moreover, the Finance
Act, of 2017 also removed any official contribution limits on companies. In
other words, an individual or a company can donate as much as they want to
a political party. Hence statement 1 is correct and statement 2 is not
correct.

Q 41.A

● Just Energy Transition Partnerships (JETP) aim to bridge the gap between
developed and developing nations in moving towards clean energy.
Essentially, JETP is a financing mechanism. In a Partnership, wealthier
nations fund a coal-dependent developing nation to support the country’s own
path to phase out coal and transition towards clean energy while addressing
the social consequences. Hence, statement 1 is correct.
● JETP funding can go through grants, loans, or investments. As of March
2023, the donor pool includes the International Partners Group (IPG) and the
Glasgow Financial Alliance for Net Zero (GFANZ) Working Group. The IPG
consists of Japan, the USA, Canada, Denmark, France, Germany, Italy,
Norway, the EU, and the UK. The GFANZ Working Group comprises
multilateral and national development banks and finance agencies such as
HSBC and Citi Bank.
● The first Just Energy Transition Partnership was with South Africa and was
announced at COP 26 Glasgow in November 2021. The funders, five of the
current IPG members, pledged 8.5 billion USD in the first financing round. A
year later, at COP 27 Sharm El Sheikh, South Africa published its JETP
Implementation Plan (JETP IP). This JETP is expected to prevent up to 1-1.5
gigatons of emissions from the atmosphere over the next 20 years. Hence,
statement 2 is not correct.
Arun Singh Tomar
● The second Partnership was announced at the G20 Bali Summit in November
2022. Indonesia is set to receive an initial 20 billion USD in public and private
financing over the next three to five years. The donors will assist via grants,
concessional loans, market-rate loans, guarantees, private investments, and
technical assistance. In February 2023, Indonesia launched the Secretariat for
the Just Energy Transition Partnership.
● The next one was Vietnam. The Vietnam JETP was announced in December
2022 after a lengthy negotiation process. This Partnership will assist Vietnam
in finance, technology, and capacity building. It will also support the country’s
policy and regulation improvement to increase private investment in
renewable energy. Hence, statement 3 is not correct.

Q 42.A

● Indian Army successfully test-fired rocket and turret guns of indigenous Light
Combat Helicopter (LCH) Prachand (means fierce).
● It was inducted into Indian Air Force in 2022.
● LCH Prachand is a multi-role combat helicopter, designed and developed by
Hindustan Aeronautics Ltd.
● Only attack helicopter in the world that can land and take off at an altitude of
5,000 metres.
● Capable of firing air-to-ground and air-to-air missiles.
● Fitted with 5.8-tonne twin-engine named Shakti engine, primarily designed for
deployment in highaltitude areas (like Siachen glacier).
● It has best stealth features, armored-shield systems, and dark-mode attack
capability.
● Hence option (a) is the correct answer.

Q 43.B

• The ‘’Vajra Mushti Kalaga’’ is a form of wrestling different from conventional


grappling and entails two jettys taking a swipe at each other’s head with a
knuckleduster.
• The fight is real and the jettys make all efforts to draw blood from the
opponent’s head and a referee intervenes on noticing the first drop.

Arun Singh Tomar


• Whosoever draws the blood from the opponent’s head first is declared the
winner. Though this form of wrestling was popular during the period of the
Vijayanagar rulers who reigned between the 14th and the 17th centuries, it has
gone extinct and takes place only during Dasara. It is not associated with the
Vajrayana form of Buddhism.
• Medieval travelers from the Portuguese noticed this form of wrestling during
the Navaratri celebrations in the Vijayanagar empire and have left detailed
accounts of it. Historically this form of martial art was popular and has a hoary
past but has been on a decline and has gone out of vogue in modern times.
• Hence option (b) is the correct answer.
Q 44.A

● The Mouvement Du 23 Mars (M23) is an armed group operating in the


Democratic Republic of the Congo (DRC). The prominent rebel groups in the
DRC, besides M23 include the Allied Democratic Forces (ADF) and the
Cooperative for Development of the Congo (CODECO).
● Recently, the UN International Organization of Migration (IOM) reported that
the number of people who have been internally displaced in the Democratic
Republic of Congo (DRC) has risen to 6.9 million. In the eastern province of
North Kivu, nearly a million people have been displaced due to the ongoing
conflict with the rebel group, Mouvement du 23 Mars (M23).
● The conflict in the DRC dates back to the 1990s when it went through two civil
wars in 1996 and 1998. The conflict erupted in the wake of the Rwandan
genocide in 1994 where ethnic Hutu extremists killed nearly one million
minority ethnic Tutsis and non-extremist Hutus.
● The armed groups have been supported by the governments of Rwanda,
Uganda, and Burundi at various points, acting as proxies for each country’s
interests in the region.
● Hence option (a) is the correct answer.

Q 45.B

● The National Aeronautics and Space Administration (NASA) is set to launch


the Atmospheric Waves Experiment (AWE) to study one of the important
drivers of Space weather – the Earth’s weather.

Arun Singh Tomar


● AWE is a first-of-its-kind NASA experimental attempt aimed at studying the
interactions between terrestrial and Space weather. Hence statement 1 is
correct but statement 2 is not correct.
● It is planned under NASA’s Heliophysics Explorers Program, the $42 million
mission will study the links between how waves in the lower layers of the
atmosphere impact the upper atmosphere, and thus, Space weather.
● AWE will be launched and mounted on the exterior of the Earth-orbiting
International Space Station (ISS). From the vantage point, it will look down at
the Earth and record the colorful light bands, commonly known as airglow.
Hence statement 3 is correct.
Q 46.A

● Countries reinforced their commitment to combat Transnational Organized


Crime at a Ministerial Conference in Palermo, Sicily (Italy).
● The Palermo Convention, officially known as the "United Nations Convention
against Transnational Organized Crime," is an international treaty adopted by
the United Nations in 2000. It is supplemented by three protocols that address
specific issues related to transnational organized crime. These protocols are
often referred to as the "Palermo Protocols."
● Hence option (a) is the correct answer.

Q 47.A

● The Election Commission of India has designed in-house software for


complete Candidate and election management through ‘ENCORE’ which
stands for Enabling Communications on Real-time Environment.
● This provides a seamless facility for Returning Officers to process candidate
nomination, affidavit, Voter turnout, counting, results and data management.
● The ENCORE counting application is an end-to-end application for returning
officers to digitize the votes polled, tabulate the round-wise data and then take
out various statutory reports of counting.
● Another application called ENCORE Scrutiny Application allows Returning
Officers to do scrutiny of the nominations filed by the candidates on-line.

Arun Singh Tomar


● After verification of the nomination the status is marked as Accepted,
Rejected or Withdrawn helping the Returning Officer to prepare the final list of
contesting candidates and assign the symbols.
● The National Technical Research Organisation is a technical intelligence
agency of India. It was set up in 2004. The agency reports to the National
Security Advisor and to the Prime Minister's Office.
● Hence statement 1 is correct but statement 2 is not correct.

Q 48.B

● The Indian Parliament passed the Energy Conservation (Amendment) Bill


2022, which modifies the 2001 Energy Conservation Act. The Ministry of
Power notified the Carbon Credit Trading Scheme 2023 and will soon notify
entities obligated to comply with GHG emission regulations. To ensure that
the identified entities’ percentage of total energy requirements come from
non-fossil fuel sources, the government will release the modalities based on
the Bureau of Energy Efficiency (BEE) recommendations.
● MoEFCC will notify the emission intensity target for obligated entities upon the
recommendation of the Ministry of Power. Emission intensity is the total
amount of greenhouse gas emitted for every unit of GDP.

Key Features of the CCTS 2023:


● Cap-and-trade system: The CCTS employs a cap-and-trade system, where a
cap is set on the total amount of greenhouse gas (GHG) emissions allowed
from a group of entities. Entities covered under the scheme have emission
intensity reduction targets. Entities that achieve or exceed their targets earn
carbon credit certificates, while those that fall short need to purchase credits
from the market.
● Trading mechanism: Obligated entities will earn a carbon credit certificate if
they surpass the target assigned to them. The certificate will be issued by
BEE. Then the Carbon credit certificates are traded on designated power
exchanges. Hence statement 1 is not correct.

Governance and oversight:

Arun Singh Tomar


● The National Steering Committee for the Indian Carbon Market (NSCICM)
oversees the overall functioning of the carbon market.
● The Bureau of Energy Efficiency (BEE) will act as the administrator for the
ICM, responsible for developing GHG emissions trajectory and targets for
obligated entities.
● The Grid Controller of India Limited will be the designated agency for
maintaining the ICM Registry and overseeing transactions among obligated
entities.
● The Central Electricity Regulatory Commission (CERC) will act as the
regulator for carbon credit trading. They will register power exchanges for
trading carbon credit certificates, protect buyer and seller interests, and
prevent fraud or mistrust. Hence statement 2 is correct.

Q 49.A

● Recent Context: In elections held at its assembly on Friday for the 2024–25
biennium, India was reelected to the International Maritime Organisation
(IMO) Council with the highest tally.
● IMO – the International Maritime Organization – is the United Nations
specialized agency with responsibility for the safety and security of shipping
and the prevention of marine and atmospheric pollution by ships. IMO's work
supports the UN sustainable development goals. Hence, statement 1 is
correct.
● IMO’s organizational structure consists of the IMO assembly, IMO council and
five main committees. The IMO Assembly consists of all Member States and
is the highest governing body of the Organization. It is responsible for
approving the work programme, voting on the budget and determining the
IMO’s financial arrangements.
● The IMO Council is elected by the Assembly for terms of two years. It acts as
the Executive Organ of IMO and is responsible, under the Assembly, for
supervising the work of the Organization.

Council members are categorized as:


● Category (a): 10 States with the largest interest in providing international
shipping services
Arun Singh Tomar
● Category (b): 10 States with the largest interest in international seaborne
trade (of which India is a member).
● Category (c): 20 States not elected under (a) or (b) above, which have
special interests in maritime transport or navigation and whose election to the
Council will ensure the representation of all major geographic areas of the
world
● India's re-election falls under the Category of 10 states with "the largest
interest in international seaborne trade", alongside Australia, Brazil, Canada,
France, Germany, the Netherlands, Spain, Sweden, and the United Arab
Emirates (UAE). Hence, statement 2 is not correct.
● India has been one of the earliest members of the IMO, having ratified its
Convention and joined it as a member-state in the year 1959. India has had
the privilege of being elected to and serving the Council of the IMO, ever
since it started functioning, and till date, except for two years for the period
1983-1984. Hence, statement 3 is not correct.

Q 50.B

● India, as the Incoming Support Chair of the Global Partnership on Artificial


Intelligence (GPAI), stands at the forefront, proudly hosting the annual GPAI
Summit from December 12-14, 2023, at the Bharat Mandapam in New Delhi.
● The Global Partnership on Artificial Intelligence or GPAI is an international
and multistakeholder initiative to guide the responsible development and use
of artificial intelligence consistent with human rights, fundamental freedoms,
and shared democratic values, as reflected in the OECD Recommendation on
AI. Hence, statement 1 is correct.
● GPAI’s founding members include Australia, Canada, France, Germany,
India, Italy, Japan, Mexico, New Zealand, the Republic of Korea, Singapore,
Slovenia, the United Kingdom, the United States of America, and the
European Union. Currently, It has 29 member countries.
● GPAI brings together leading experts from industry, civil society,
governments, and academia to bridge the gap between theory and practice
on AI by supporting cutting-edge research and applied activities on AI-related
priorities with the goal of fostering international cooperation.

Arun Singh Tomar


● GPAI secretariat is hosted by the Organisation for Economic Co-operation
and Development. The OECD is also a Permanent Observer to GPAI’s
governing bodies and contributes experts to participate in GPAI’s Working
Groups and the annual Multistakeholder Experts Group Plenary. Hence,
statement 2 is correct.
● The first three GPAI summits were held in Montreal, Paris and Tokyo
respectively. India, which took over the presidency of the GPAI in November
last year, will host the three-day summit in December 2023. Hence,
statement 3 is not correct.

Q 51.D

● Recently, a Lok Sabha MP was accused of cash for query scam and the Lok
Sabha Ethics Committee took up the investigation against the MP.
● A Presiding Officers’ Conference held in Delhi in 1996 first mooted the idea of
ethics panels for the two Houses. Then Vice President (and Rajya Sabha
Chairman) K R Narayanan constituted the Ethics Committee of the Upper
House on March 4, 1997, and it was inaugurated that May to oversee the
moral and ethical conduct of members and examine cases of misconduct
referred to it. The Rules applicable to the Committee of Privileges also apply
to the ethics panel. Hence, statement 1 is not correct.
● Any person can complain against a Member through another Lok Sabha MP,
along with evidence of the alleged misconduct, and an affidavit stating that the
complaint is not “false, frivolous, or vexatious”. If the Member himself
complains, the affidavit is not needed. The Committee does not entertain
complaints based only on media reports or on matters that are sub judice. The
Committee makes a prima facie inquiry before deciding to examine a
complaint. It makes its recommendations after evaluating the complaint. The
Committee presents its report to the Speaker, who asks the House if the
report should be taken up for consideration. There is also a provision for a
half-hour discussion on the report. Hence, statement 2 is not correct.

Arun Singh Tomar


● The work of the Ethics Committee and the Privileges Committee often
overlap. An allegation of corruption against an MP can be sent to either body,
but usually, more serious accusations go to the Privileges Committee. The
Ethics Committee can take up only cases of misconduct that involve MPs.

Q 52.C

● The blend between diesel and ethanol (also called “diesohol”) has been
considered as one of the candidate alternative fuels for diesel substitution.
The use of ethanol blended with diesel fuel in a compression ignition engine
has some potential for exhaust black smoke reduction. Hence, statement 1
is correct.
● Doping of ethanol with petrol supplies extra oxygen for complete combustion,
which reduces carbon monoxide levels in auto emissions and therefore, it is
considered more environment friendly as it lessens air pollution.
● Hindustan Petroleum Corporation Limited (HPCL) has successfully launched
a groundbreaking pilot study on vehicles using E27 fuel and Ethanol Blended
Diesel Fuel. Under the aegis of MoP&NG, HPCL has become the first Oil
Marketing Company in India to initiate such a comprehensive research
program, in line with the "Roadmap for Ethanol Blending in India by 2025,"
which aims to promote the adoption of Ethanol Blending in gasoline. Hence,
statement 2 is correct.

Q 53.B

● Recently, the Geological Survey of India discovered lithium reserves in the


Reasi district of Jammu and Kashmir. Hence, statement 2 is correct.
● Argentina, together with Chile and Bolivia, form the so-called “Lithium
Triangle” and is currently the fourth largest producer. It has the world’s third-
largest lithium reserve. Hence, statement 1 is correct.
● India imports all major components that go into lithium-ion cell manufacturing.
The country’s lithium-ion import bill for FY23 was ₹23,171 crore. In FY22,
imports for lithium-ion were ₹13,673.15 crore.
● KABIL is a joint venture between the National Aluminium Company (NALCO),
Mineral Exploration Corporation Ltd. (MECL), and Hindustan Copper Ltd.
(HCL). It was formed for the identification, acquisition, development, and

Arun Singh Tomar


processing of strategic minerals overseas for use in India. Hence, statement
3 is not correct.
● So far, India has always been import-dependent, not only for lithium but also
for other minerals such as nickel and cobalt. Now, to reduce dependency on
imports and grow further in the EV manufacturing space, experts reckon the
government has lifted the ban on lithium mining.
● Hence option (b) is the correct answer.

Q 54.D

● Since 2000, the International Organization for Migration (IOM) has been
producing its flagship world migration reports every two years. Hence,
statement 1 is not correct.
● The World Migration Report 2022, the eleventh in the World Migration Report
series, has been produced to contribute to an increased understanding of
migration and mobility throughout the world
● This new edition presents key data and information on migration as well as
thematic chapters on highly topical migration issues, and is structured to focus
on two key contributions for readers:
● Part I: key information on migration and migrants (including migration-related
statistics); and
● Part II: balanced, evidence-based analysis of complex and emerging
migration issues.
● This flagship World Migration Report has been produced in line with IOM’s
Environment Policy and is available online only. Hence, statement 2 is not
correct.
● Printed hard copies have not been made in order to reduce paper, printing
and transportation impacts.
● Hence, option (d) is the correct answer.

Q 55.C

Arun Singh Tomar


● GBA is an India-led Initiative to develop an alliance of Governments,
International organisations and Industry to facilitate adoption of biofuels
through international cooperation. Hence statement 1 is correct.
● The Global Biofuels Alliance (GBA) was formally launched by the Indian PM
on the side-lines of the G20 Summit in New Delhi. Hence statement 2 is
correct.
Q 56.C

● Water pollution caused by detergents is now a big concern in the global


context. The per capita detergent consumption in India is around 2.7 kilogram
per year. It is around 3.7 kg in the Philippines and Malaysia and 10 kg in the
United States of America.
● Nonylphenol is commonly used in the production of Nonylphenol Ethoxylates
(NPEs). NPEs are used as surfactants as well as in day-to-day consumer
products such as detergents, wetting agents and dispersants. It bio-
accumulates and can pose serious environmental and health risks. It is a
persistent, toxic, bio-accumulative chemical that acts as an endocrine
disruptor. Hence, statements 1 and 2 are correct.
● NPEs enter the environment and ultimately break down to nonylphenols that
can enter different environmental matrices such as water and soil. The
chemical can also possibly be released during industrial cleaning processes
and from wastewater produced during the production of NPEs.
● It has been detected in human breast milk, blood and urine, and is associated
with reproductive and developmental effects in rodents. It is recommended to
find substitutes of nonylphenol. The United Nations Environment Programme
(UNEP) has also designated nonylphenol as a chemical of global concern.
Hence, statement 3 is correct.
● In India, the Bureau of Indian Standards (BIS) has set standards for phenolic
compounds in drinking water (1 ppb) and surface water (5 parts per million
(ppm)). However, at present, there are no standards exclusively for
nonylphenols in drinking and surface waters in India. Hence, statement 4 is
not correct.

Q 57.D

Arun Singh Tomar


● JT-60SA, the world's largest and most advanced nuclear fusion reactor
started its official operations in Japan's Ibaraki Prefecture.
● The JT-60SA, a six-story-high tokamak, is designed to contain and control
plasma heated to a staggering 200 million degrees Celsius.
● The objective is to explore the viability of fusion as a secure, expansive, and
carbon-neutral net energy source, where the energy produced surpasses is
more than the input required for its production.
● This joint venture between the European Union and Japan serves as a
precursor to the International Thermonuclear Experimental Reactor (ITER)
currently under construction in France. Both projects share the ambitious goal
of achieving net energy gain from fusion, a milestone that could revolutionize
our energy systems. Hence, option (d) is the correct answer.
● The experimental nuclear fusion reactor is quite a new technology. However,
some say that it is the answer to humanity's future energy needs by
establishing nuclear fusion as a clean energy source.
● Unlike traditional nuclear fission used in power plants, which splits atomic
nuclei, fusion mimics the sun's energy production by merging two nuclei,
potentially offering a safer and more abundant energy source.
● Fusion energy stands out for its safety advantages over fission, as it carries
no risk of catastrophic accidents like the Fukushima disaster in 2011 and
generates minimal radioactive waste.

Q 58.C

• Australians recently voted in a referendum to decide whether the country’s


indigenous peoples should be formally consulted in making laws. The
referendum question read: “A Proposed Law: to alter the Constitution to
recognize the First Peoples of Australia by establishing an Aboriginal and
Torres Strait Islander Voice. Do you approve this proposed alteration?”
Hence, option (c) is the correct answer.
• The expression, as well as the word ‘aboriginal’, refers to the indigenous
inhabitants of the continent — people who lived on the Australian mainland
and surrounding islands for tens of thousands of years before the first
Europeans arrived in the early 17th century.

Arun Singh Tomar


• The Torres Strait Islands, mentioned in the referendum question, is an
archipelago of small islands in the Torres Strait, a narrow body of water
between the northern tip of the state of Queensland and the large island of
Papua New Guinea.
Q 59.D

● NASA launched a new sounding rocket to study a stellar event in the Cygnus
Loop, which is a supernova remnant that took place 20,000 years ago.
● Cygnus is about 2,600 light years away from the Earth. The sounding rocket
mission, named the Integral Field Ultraviolet Spectroscope Experiment
(INFUSE) was launched from the White Sands Missile Range in New Mexico.
Hence, option (d) is the correct answer.
Q 60.A

● Solar power projects can be set up anywhere in the country, however, the
solar power projects developed in a scattered manner lead to higher project
cost per MW and higher transmission losses. Individual projects of smaller
capacity incur significant expenses in site development, drawing separate
transmission lines to nearest substation, procuring water and in creating of
other necessary infrastructure. It also takes longer time for project developers
to acquire land, get all types of clearances and permissions, etc. which
ultimately delays the project.
● To overcome these challenges, the scheme for “Development of Solar Parks
and Ultra-Mega Solar Power Projects” was rolled out in December 2014 with
the objective of facilitating the solar project developers to set up projects
expeditiously. Under this scheme, the Government has sanctioned 50 Solar
Parks with an aggregate capacity of 37,990 MW in 12 States across the
country, so far. Against this sanction, 11 Solar Parks with an aggregate
capacity of 8521 MW have been completed and 7 Solar Parks with an
aggregate capacity of 3985 MW have been partially completed. In these
parks, solar projects of an aggregate capacity of 10,237 MW have been
developed. Hence, statement 1 is not correct.

Salient features:

Arun Singh Tomar


● The scheme envisages supporting the States/UTs in setting up solar parks at
various locations in the country with a view to creating the required
infrastructure for setting up of solar power projects. The solar parks provide
suitable developed land with all clearances, transmission systems, water
access, road connectivity, communication networks, etc. The scheme
facilitates and speeds up the installation of grid-connected solar power
projects for electricity generation on a large scale.
● All the States and Union Territories are eligible for getting benefits under the
scheme.
● The capacity of the solar parks shall be 500 MW and above. However, smaller
parks are also considered where contiguous land may be difficult to acquire in
view of difficult terrain and where there is an acute shortage of non-
agricultural land.
● The Solar Parks are developed in collaboration with the State Governments
and their agencies, CPSUs, and private entrepreneurs. The implementing
agency is termed as Solar Power Park Developer (SPPD). There are 8 modes
for selection of SPPDs. Solar Energy Corporation of India (SECI) acts as a
nodal agency for the scheme. Hence, statement 2 is correct.
● Recently, the Union Ministry of New and Renewable Energy has extended the
scheme for ‘Development of Solar Parks and Ultra Mega Solar Park Projects’
till FY26 because of delays in implementation. The reasons for the delay in
establishing solar parks include challenges in the acquisition of clear land,
mismatch in timelines between solar projects and power evacuation
infrastructure, environmental issues like the Great Indian Bustard (GIB) issue,
regulatory challenges like non-approval of solar tariff by SERCs, etc. Hence,
statement 3 is not correct.

Q 61.A

● Recently, the UN Secretary-General Antonio Guterres invoked Article 99, a


rarely used provision of the UN Charter, to bring to the attention of the
Security Council “hostilities in Gaza and Israel’ as he believes it ‘may
aggravate existing threats to international peace and security”. Hence option
(a) is the correct answer.
Q 62.A
Arun Singh Tomar
● The ‘Chief Economists Outlook’ report, which was released recently by the
World Economic Forum, aims to provide a brief overview of the current
economic situation and highlight key areas where policymakers and business
leaders should focus their efforts in response to the shocks caused by geo-
economic and geopolitical events. Hence option (a) is the correct answer.

Q 63.B

United Nations Population Fund:


● It is a subsidiary organ of the UN General Assembly and works as a sexual
and reproductive health agency.
● The UN Economic and Social Council (ECOSOC) establishes its mandate.
Hence, statement 3 is correct.
● It was established as a trust fund in 1967 and began operations in 1969.
● In 1987, it was officially renamed the United Nations Population Fund but the
original abbreviation, ‘UNFPA’ for the United Nations Fund for Population
Activities was retained.

Aim:
● UNFPA works for Sustainable Development Goals on health (SDG3),
education (SDG4) and gender equality (SDG5).
● Our goal is to end the unmet need for family planning, preventable maternal
death, and gender-based violence and harmful practices including child
marriage and female genital mutilation by 2030. Hence, statement 1 is
correct.
● UNFPA is entirely supported by voluntary contributions of donor governments,
intergovernmental organizations, the private sector and foundations and
individuals, not by the United Nations' regular budget. Hence, statement 2 is
not correct.
● Hence, option (b) is the correct answer.

Q 64.A

Arun Singh Tomar


● India has been unanimously elected as a member representing the Asian
region in the executive committee of the Codex Alimentarius Commission
(CAC), the food safety and quality standard-setting body created by the UN,
at its meeting in Rome.
● CAC is an international food standards, guidelines and codes of practice
contribute to the safety, quality and fairness of this international food trade.
Consumers can trust the safety and quality of the food products they buy and
importers can trust that the food they ordered will be in accordance with their
specifications. Hence, statement 1 is correct.
● The Codex Alimentarius commission was created by the Food and Agriculture
Organization of the United Nations and World Health Organization. Hence,
statement 2 is not correct.
Q 65.A

● Researchers have discovered an extremely high-energy particle that came


from a part of the universe where there is nothing. It was named "Amaterasu"
after a Japanese goddess. The Amaterasu particle has an energy exceeding
240 exa-electron volts (EeV). That is millions of times more powerful than the
particles produced by the Large Hadron Collider, which is the most powerful
accelerator ever built. It is second only to the “Oh-My-God” particle, another
high-energy cosmic ray detected in 1991. That came in at 320 EeV. Hence,
statement 1 is correct.
● It is common for Earth to receive low-energy cosmic ray, but those with
extremely high energy come rarely. They are thought to travel to Earth from
other galaxies and sources beyond our Milky Way. When the researchers
tried to find the source of the energy spike, they found nothing, according to
Nature. Ultrahigh energy particles like Amaterasu usually travel through space
quite smoothly since they don’t bounce off magnetic fields, like low-energy
cosmic rays. Technically, this should make it easier to pinpoint the location
where it came from, but that was not the case. Hence, statement 2 is not
correct.

Q 66.A

Arun Singh Tomar


● The Central Bureau of Investigation (CBI), as part of its ‘Chakra-II’ operation,
has achieved a major breakthrough in two more cases involving an
international online investment fraud worth hundreds of crores targeting
Indians; and a cyber-enabled impersonation racket in which Singaporean
citizens were cheated.
● The first case was registered by the agency last year, based on inputs from
various sources, including the Indian Cyber Crime Coordination Centre (I4C)
under the Ministry of Home Affairs. As alleged, in conspiracy with local
associates, the accused based overseas cheated Indian citizens on the
pretext of investment, loans, and job opportunities.
● The Central Bureau of Investigation (CBI) has launched Operation Chakra-II
to fight against transnational organized cyber-enabled financial crimes in
India. For this, India’s federal agency has partnered with Microsoft and
Amazon as well as with national and international agencies to combat and
dismantle infrastructure of illegal call centers. Hence, option (a) is the
correct answer.
Q 67.D

● PM Vishwakarma, a Central Sector Scheme was launched recently by the


Prime Minister to provide end-to-end support to artisans and craftspeople who
work with their hands and tools.
● Ministry of Micro, Small & Medium Enterprise is the nodal ministry of PM
Vishwakarma Yojana.
● The Scheme envisages provisioning of the following benefits to the artisans
and crafts persons: Recognition of artisans and craftspeople through PM
Vishwakarma certificate and ID card.
● Credit Support: Collateral free 'Enterprise Development Loans' of upto Rs. 3
lakh in two tranches of Rs. 1 lakh and Rs. 2 lakh with tenures of 18 months
and 30 months, respectively, at a concessional rate of interest fixed at 5%,
with Government of India subvention to the extent of 8%.

Eligibility criteria for the scheme:


● An artisan or craftsperson working with hands tools and engaged in one of the
family-based traditional trades in unorganized sector

Arun Singh Tomar


● Minimum age of the beneficiary should be 18 years.
● The registration and benefits under the scheme shall be restricted to
one member of the family.
● For availing benefits under the scheme, a family is defined as consisting the
husband, wife and un unmarried children.
● Hence, option (d) is the correct answer.

Q 68.D

● The state government of Gujarat has banned the plantation of Conocarpus


erectus trees, an exotic mangrove species, in both forest and non-forest areas
due to their adverse impacts on the environment and human health. The
decision comes in response to research reports highlighting the negative
effects of Conocarpus trees, including the spread of pollen that can cause
diseases such as colds, coughs, asthma, and allergies. Conocarpus is an
evergreen species with dark-green shiny leaves.
● It grows rapidly and is not preferred by wildlife or domesticated animals.
Conocarpus trees offer several benefits, but as with any plant species, there
are also potential drawbacks that need to be considered. Proper planning and
management are essential to maximize the advantages of these trees while
minimizing their negative impacts. It has been used by various public
authorities in India as landscaping for road medians, along roads, and in
public gardens.
● This tree is famous for its dark green leaves color throughout the year and
withstands harsh environmental conditions such as high and low
temperatures. It is very adaptive and could grow even in areas with extreme
salinity. It absorbs more water from soil than other species and is a threat to
groundwater.
● Hence, option (d) is the correct answer.

Q 69.C

● The first INTERPOL operation named ‘Operation Storm Makers II’ specifically
targeting the phenomenon of human trafficking-fuelled fraud has revealed
further evidence that the crime trend is expanding beyond Southeast Asia.
Arun Singh Tomar
● Many of the hotspots are regularly used to traffic victims to notorious cyber
scam centers in Southeast Asia. Victims are often lured through fake job ads
and forced to commit online fraud on an industrial scale while enduring abject
physical abuse. Fraud schemes include fake cryptocurrency investments, as
well as work-from-home, lottery and online gambling scams.
● According to Interpol, the Telangana police registered one of the first cases in
India of human trafficking committed for the purpose of forcing victims to
commit cyber fraud.
● Indian enforcement agencies also participated in the exercise. Law
enforcement agencies in 27 countries across Asia and other regions were
mobilized for the operation.
● About INTERPOL: International Criminal Police Organization (INTERPOL) is
an inter-governmental organisation (not a UN agency) comprising 196
member countries (including India), which helps police forces in all these
countries to better coordinate their actions
● Hence, option (c) is the correct answer

Q 70.C

● Indian Space Research Organisation (ISRO) launched the Aditya-L1 mission.


Aditya-L1 is India's first dedicated mission to study the Sun. The primary
objective of the mission is to observe the Sun's outermost layer, known as the
corona, and study various phenomena like solar winds and magnetic fields
associated with the Sun.
● The Sun is composed of several layers, each with distinct characteristics and
functions. These layers, from the interior to the outermost layer, are:
● Core: The core is the innermost layer and is where nuclear fusion occurs.
Here, intense heat and pressure cause hydrogen atoms to fuse into helium,
releasing an enormous amount of energy in the form of light and heat.
● Radiative Zone: Here energy produced in the core travels outward in the form
of photons of light.
● Convection Zone: In this layer, energy is transported by the rising and falling
of hot and cool material. Hot plasma rises toward the surface, cools as it
releases energy, and then descends back into the interior.

Arun Singh Tomar


● Photosphere: The photosphere is the visible surface of the Sun that emits
light and heat. It is the layer from which sunlight is directly radiated into space.
● Chromosphere: It is characterized by a reddish glow and is typically seen
during a solar eclipse.
● Transition Region: The transition region is a narrow layer between the
chromosphere and the corona. It is an area of rapid temperature increase.
● Corona: The outermost layer is the corona, an extremely hot and tenuous
outer atmosphere that extends into space. The corona is visible during a solar
eclipse as a faint halo of plasma around the Sun.
● Hence 2-1-3-5-4 is the correct sequence.
● Hence option (c) is the correct answer.

Q 71.A

● Biotransformation technology is a novel approach to ensure plastics that


escape refuse streams are processed efficiently and broken down. The tech
was co-developed by the Imperial College in London, UK, and a Britain-based
startup, Polymateria.
● Plastics made using this technology are given a pre-programmed time during
which the manufactured material looks and feels like conventional plastics
without compromising on quality. Once the product expires and is exposed to
the external environment, it self-destructs and biotransforms into bioavailable
wax. This wax is then consumed by microorganisms, converting waste into
water, CO2, and biomass. Hence, option (a) is the correct answer.
Q 72.B

● Sixty-three countries, including the US, Canada, and Kenya, signed up to the
world’s first-ever pledge to drastically cut cooling emissions at the recently
concluded COP28 climate summit
● An initiative of the United Arab Emirates as host of the 2023 United Nations
Climate Change Conference (COP28), the Pledge is one of nine non-
negotiated declarations, pledges, and charters that constitute key outcomes
for the COP28 Presidential Action Agenda. Hence, statement 1 is correct.
● It aims to raise ambition and international cooperation through collective
global targets to reduce cooling related emissions by 68% by 2050 compared

Arun Singh Tomar


to 2022 levels, significantly increase access to sustainable cooling by 2030,
and increase the global average efficiency of new air conditioners by 50%.
The emission targets draw on the modelling from the UNEP Cool Coalition
report Global Cooling Watch 2023 Keeping it Chill: How to meet cooling
demands while cutting emissions. Hence, statement 2 is correct.
● India did not join the pledge as it faces a unique challenge in balancing its
aspirations for development and environmental sustainability. A hot country
already, climate projections indicate an up to sixfold increase in the frequency
of severe heat waves by 2060. It poses a significant threat to its growing and
vulnerable population.
● It also has raised concerns about the principle of equity, emphasizing that
developed countries, with their greater historical responsibility for greenhouse
gas emissions, should bear a larger burden in emissions reduction efforts.
Hence, statement 3 is not correct.

Q 73.C

● Almost $7 trillion is invested by countries every year in subsidies and private


investments that have a direct negative impact on nature, according to a new
United Nations report 'State of Finance for Nature'. The figure is equivalent to
7 percent of the global gross domestic product.
● The report, presented by the UN Environment Portal (UNEP) at the 28th
Conference of Parties (COP28) to the United Nations Framework Convention
on Climate Change, analyses and quantifies the private finance flows that
directly affect nature negatively for the first time. Hence, option (c) is the
correct answer.
Q 74.C

● A five-judge Constitution bench of the Supreme Court, on December 5, began


hearing pleas challenging Section 6A of the Citizenship Act, which was
introduced in the statute following the signing of the Assam Accord.
● Section 6A of the Citizenship Act is a special provision on the citizenship of
persons covered by the Assam Accord. Under this section, people who
entered India between January 1, 1966, and March 25, 1971, and have been

Arun Singh Tomar


living in Assam, will be allowed to register themselves as citizens. Hence,
statement 1 is correct.
● Section 6A was inserted into the 1955 Citizenship Act in December 1985.
Assam is the only state in India to have an exclusive cut-off date for
citizenship. For the rest of the country, the cut-off date is July 19, 1949.
Hence, statement 2 is not correct but statement 3 is correct.

Arun Singh Tomar


Arun Singh Tomar

You might also like